DSAT Reading Words in Context Practice Questions #2

1. Up to this moment the young earl had stood still, as if spell-bound; but being now convinced that the spirit had fled, he pressed forward, and, ere many seconds, emerged from the brake. The full moon was rising as he ______________ and illuminating the glades and vistas, and the calmness and beauty of all around seemed at total variance with the fearful vision he had just witnessed.

Which of the following choices best completes the text with the most logical and precise phrase?
A. went from,
B. issued forth,
C. cropped up,
D. came across,

2. They walked on in silence, for the earl could not help dwelling upon the vision he had witnessed, and his companion appeared equally ____________ In this sort they descended the hill near Henry the Eighth’s Gate, and entered Thames Street.

Which of the following choices best completes the text with the most logical and precise word?
A. unconcerned.
B. worried.
C. abstracted.
D. alive.

3. Turning off on the left into the lower road, ________ around the north of the castle, and following the course of the river to Datchet, by which it was understood the royal cavalcade would make its approach, the procession arrived at an open space by the side of the river, where it came to a halt.

Which of the following choices best completes the text with the most logical and precise word?
A. dressing
B. skirting
C. avoiding
D. leaving

4. Presently the sound of trumpets smote his ear, and a numerous and splendid retinue was seen advancing, consisting of nobles, knights, esquires, and gentlemen, ranged according to their degrees, and all sumptuously ___________ in cloths of gold and silver, and velvets of various colors, richly embroidered.

Which of the following choices best completes the text with the most logical and precise word?
A. worn
B. addressed
C. redressed
D. appareled

5. The person who thus spoke then stepped forward, and threw a glance so full of significance at Anne Boleyn that she did not care to dispute the order, but, on the contrary, laughingly ______________ it.

Which of the following choices best completes the text with the most logical and precise word or phrase?
A. railed to
B. railed against
C. acquiesced against
D. acquiesced to

6. The favor in which he stood with his royal master procured him admittance to his presence at all hours and at all seasons, and his influence, though seldom exerted, was very great. He was especially serviceable in turning aside the edge of the king’s displeasure, and more frequently exerted himself to _______ the storm than to raise it.

Which of the following choices best completes the text with the most logical and precise word?
A. allay
B. allow
C. align
D. stoke

7. He took her hand, and led her to the upper part of the chamber, where two chairs of state were set beneath a canopy of crimson velvet embroidered with the royal arms, and placed her in the seat hitherto allotted to the previous queen. A smile of triumph irradiated Anne’s lovely countenance at this ________________ nor was her satisfaction diminished as Henry turned to address the assemblage.

Which of the following choices best completes the text with the most logical and precise phrase?
A. show of force,
B. mark of distinction,
C. level of crime,
D. morbid scene,

8. This ghostly rider wore the antlered helmet described by Surrey, and seemed to be habited in a garb of deer-skins. Before him flew a large owl, and a couple of great black dogs ran beside him. Staring in _____________ wonder at the sight, the two youths watched the mysterious being scour a glade brightly illumined by the moon, until, reaching the pales marking the confines of the Home Park, he leaped them and disappeared.

Which of the following choices best completes the text with the most logical and precise word?
A. speechless
B. unspeakable
C. amazed
D. lonely

9. Before they reached the hill, at the end of the long avenue, a heavy thunderstorm came on, and the lightning, playing among the trees, seemed to reveal a thousand fantastic forms to their half-blinded gaze. Presently the rain began to descend in torrents, and compelled them to ___________beneath a large beech-tree.

Which of the following choices best completes the text with the most logical and precise phrase?
A. abide beneath
B. stay wet
C. align themselves
D. take refuge

10. A verdant path, partly beneath the trees, and partly on the side of the lake, led Wolsey to the forester’s hut. Constructed of wood and clay, with a thatched roof, green with moss, and half overgrown with ivy, the little building was in __________ keeping with the surrounding scenery.

Which of the following choices best completes the text with the most logical and precise word?
A. admiring
B. admirable
C. poor
D. abominable

  1. B. The young earl “emerged from the brake”. We are looking for a word that would fit the context of the sentence and describe his emerging. Answer option A does not fit the structure of the sentence as there is no further description of where he went from. Answer C implies something slowly sneaking up over time, which is not the proper in this context. Answer D does not mean “emerging” and is therefore incorrect. This only leaves answer option B, that he issued forth (meaning “emerged”).
  2. C. The earl is dwelling on the vision, not on his companion or on making conversation. He is distracted by what they have seen. His companion, then, is equally distracted. Abstracted is a synonym for distracted, making option C the best answer. Answer A is the opposite of the correct answer. Answer B is incorrect as there is no evidence the earl is worried, only that he is distracted and thinking about the vision. There is no discussion of them feeling or being alive or dead, making answer option D incorrect.
  3. B. Given that the blank is followed by the word “around” we must choose an answer that can be done “around the north of the castle”. A procession could not dress around a castle, nor could it avoid or leave around it. This leaves answer B as the correct answer. To “skirt around” means to go around the edges of something.
  4. D.  The context is describing what the gentlemen are wearing. This makes options B and C incorrect as they are both words that look similar to the word “dressed” but have different meanings. “Worn” does not fit the context of the sentence. This leaves answer option D. The men are “sumptuously appareled in cloths”. Appareled is a synonym for “dressed”.
  5. D. We see in the first part of the sentence that Anne is not going to dispute (go against) the order. The author then sets up a contrasting statement by saying “but, on the contrary”. We therefore need to pick an answer that means the opposite of “dispute”. To acquiesce means to give in. The appropriate preposition for acquiesce is “to”. This makes option D correct. You cannot acquiesce against something, making option C incorrect and options A and B would not show contrast with “dispute” since “to rail against” means to speak aggressively against something.
  6. A. We learn in the first sentence that this man is favored by the king and has great influence over the king. The second sentence shows us how the man uses that influence to turn “aside the edge of the king’s displeasure.” The final phrase sets up a contrast. Rather than raising the storm of anger he is more likely to subdue it. This is the meaning of the word “allay” (related to the word alleviate) which makes answer option A correct. Answer options B and D mean close to the opposite of the author’s intended meaning. Answer option C is incorrect as “align” is not something that can be done with storms or anger.
  7. B. Anne is taken to sit in the seat of the queen this is a good thing for Anne and elicits from her a “smile of triumph”, making options C and D incorrect. Option A is incorrect as there is no evidence that Henry has shown any force toward her. Instead, being sat in the queen’s chair is a “mark of distinction” setting her apart from anyone else who might be watching.
  8. A. The boys are watching a ghostly rider, which makes their wonder understandable. They are together and so option D doesn’t fit. Their wonder isn’t unspeakable, a word for things so horrible they can’t be said. “Amazed wonder” would be redundant. This leaves option A. They are speechless with wonder.
  9. D. Given the storm, the people need to find a safe place. They “take refuge” under a tree to escape the storm. They are not living beneath the tree, making option A incorrect. They would not want to stay wet, but rather, dry making option B incorrect. They are not aligning themselves with anything, making option C incorrect.
  10. B. We see that the house is beneath trees, and on the side of the lake. It is made of natural materials and therefore matches the scenery. This makes options C and D incorrect as they would imply that the house does not match the scenery. The house is not admiring, making option A incorrect.


    If you would like to expand your vocabulary by reading of the story of Anne, Henry, and ghostly riders, the whole book is available for free through Project Gutenberg:
    https://www.gutenberg.org/cache/epub/2866/pg2866-images.html

Digital SAT Practice Questions: Command of Evidence

1. The following is an excerpt from Essays by Ralph Waldo Emerson:

The farmer imagines power and place are fine things. But the President has paid dear for his White House.  It has commonly cost him all his peace, and the best of his many attributes. To preserve for a short time so conspicuous an appearance before the world, he is content to eat dust before the real masters who stand erect behind the throne. Or, do men desire the more substantial and permanent grandeur of genius? Neither has this an immunity. He who by force of will or of thought, is great, and overlooks thousands, has the charges of that eminence. With every influx of light comes new danger.

Based on the passage, which of the following would best describe the author’s attitude toward power?
A. Power is something to which all men strive and few men attain. Power once found will always corrupt.
B. While men see power as something laudable and to be sought after, they forget that it comes at a steep price.
C. Power should be avoided by all men who can stay away from it. Each man should be content where he is.
D. Power is all well and fine for the president and similar lofty persons, but is not suitable for the common farmer.

2.  The following is an excerpt from Essays by Ralph Waldo Emerson:

The scholar of the first age received into him the world around; brooded thereon; gave it the new arrangement of his own mind, and uttered it again. It came into him life; it went out from him truth. It came to him short-lived actions; it went out from him immortal thoughts. It came to him business; it went from him poetry. It was dead fact; now, it is quick thought. It can stand, and it can go. It now endures, it now flies, it now inspires. Precisely in proportion to the depth of mind from which it issued, so high does it soar, so long does it sing.

Based on the passage, what is the “it” referred to throughout the paragraph?
A. Poetry
B. Music
C. The world
D. Actions

3. The following is an excerpt from Essays by Ralph Waldo Emerson:

Society everywhere is in conspiracy against the manhood of every one of its members. Society is a joint-stock company, in which the members agree, for the better securing of his bread to each shareholder, to surrender the liberty and culture of the eater. The virtue in most request is conformity. Self-reliance is its aversion. It loves not realities and creators, but names and customs.

According to the text, what does society provide in return for the surrender of liberty and culture?
A. Food security
B. Names and customs.
C. Stocks
D. Manhood.

4. The following is adapted from Essays by Ralph Waldo Emerson:

We have a great deal more kindness than is ever spoken. Barring all the selfishness that chills like east winds the world, the whole human family is bathed with an element of love like a fine ether. How many persons we meet in houses, whom we scarcely speak to, whom yet we honor, and who honor us! How many we see in the street, or sit with in church, whom, though silently, we warmly rejoice to be with! The effect of the indulgence of this human affection is a certain cordial exhilaration. The emotions of benevolence and complacency which are felt toward others, are likened to the material effects of fire; so swift, or much more swift, more active, more cheering are these fine inward irradiations. From the highest degree of passionate love, to the lowest degree of good will, they make the sweetness of life.

Based on the passage, what is the general approach of each member of society to each other member of society?
A. They treat one another with chilly selfishness.
B. When they see one another in the street or church they silently ignore them.
C. They see one another in a complacent and benevolent kindness.
D. They experience passionate love for one another.

5. The following is adapted from Essays by Ralph Waldo Emerson:

Friendship requires that rare mean betwixt likeness and unlikeness, that piques each with the presence of power and of consent in the other party. Let him not cease an instant to be himself. Better be a nettle in the side of your friend, than his echo. The condition which high friendship demands is ability to do without it. That high office requires great and sublime parts. There must be very two before there can be very one. Let it be an alliance of two large formidable natures, mutually beheld, mutually feared, before yet they recognize the deep identity which beneath these disparities unites them.

Based on the passage, what must be true in order for friendship to thrive?
A. Each friend must strive to be as much like the other as possible.
B. Each friend must be great and sublime or the friendship will perish.
C. Each friend must be wholly different from the other.
D. Each friend must be true to himself and have a mutual appreciation for the other.

6. The following is an excerpt from Essays by Ralph Waldo Emerson:

A plentiful fortune is reckoned necessary, in the popular judgment, to the completion of this man of the world: and it is a material deputy which walks through the dance which the first has led. Money is not essential, but this wide affinity is, which transcends the habits of clique and caste, and makes itself felt by men of all classes. If the aristocrat is only valid in fashionable circles, and not with truckmen, he will never be a leader in fashion; and if the man of the people cannot speak on equal terms with the gentleman, so that the gentleman shall perceive that he is already really of his own order, he is not to be feared.

According to the passage, what needs to be true in order for men to be a complete man of the world?
A. He must be wealthy and able to use his fortune well.
B. He must be humble, regardless of his wealth.
C. He must be able to connect with men of all social classes.
D. He must be fashionable and a gentleman.

7. The following is an adaptation from Essays by Ralph Waldo Emerson. In it, Emerson discusses the works of William Shakespeare:

His lyric power lies in the genius of the piece. The sonnets, though their excellence is lost in the splendor of the dramas, are as inimitable as they: and it is not a merit of lines, but a total merit of the piece; like the tone of voice of some incomparable person, so is this a speech of poetic beings, and any clause as unproducible now as a whole poem. Though the speeches in the plays, and single lines, have a beauty which tempts the ear to pause on them for their euphuism, yet the sentence is so loaded with meaning, and so linked with its foregoers and followers, that the logician is satisfied. His means are as admirable as his ends; every subordinate invention, by which he helps himself to connect some irreconcilable opposites, is a poem too.

According to the text, what makes the work of Shakespeare so admirable?
A. It is part of a greater body of works by English authors.
B. Each part is beautiful on its own and as a part of a beautiful whole.
C. Shakespeare’s tone is incomparable to that of any other writer.
D. Each line is better than the one that is written before it.

8. The following is an adaptation from Essays by Ralph Waldo Emerson:

Prudence is the virtue of the senses. It is the science of appearances. It is the outermost action of the inward life. It moves matter after the laws of matter. It is content to seek health of body by complying with physical conditions, and health of mind by the laws of the intellect. The world of the senses is a world of shows; it does not exist for itself, but has a symbolic character; and a true prudence or law of shows recognizes the co-presence of other laws and knows that its own office is subaltern; knows that it is surface and not center where it works. Prudence is false when detached. It is legitimate when it is the Natural History of the soul incarnate, when it unfolds the beauty of laws within the narrow scope of the senses.

Prudence, as expressed by the passage, exists for what reason?
A. Not for itself, but to work on the surface seeking the health of the person.
B. To seek only after appearances in order to best present the body to the world.
C. To be a symbol of good moral character when interacting with the world.
D. Not for itself, but to seek good in the world around.

9. The following is an excerpt from Essays by Ralph Waldo Emerson:

The actions and events of our childhood and youth are now matters of calmest observation. They lie like fair pictures in the air. Not so with our recent actions,—with the business which we now have in hand. On this we are quite unable to speculate. Our affections as yet circulate through it. We no more feel or know it than we feel the feet, or the hand, or the brain of our body. The new deed is yet a part of life,—remains for a time immersed in our unconscious life. In some contemplative hour it detaches itself from the life like a ripe fruit, to become a thought of the mind. Instantly it is raised, transfigured; the corruptible has put on incorruptibly. Henceforth it is an object of beauty, however base its origin and neighborhood.

Based on the passage, why are recent memories so difficult to calmly observe compared to older memories?
A. Recent memories are still filled with emotions and still part of our life.
B. Older memories have had more time to process and fade to nothingness.
C. Recent memories have detached from life like an unripe fruit and cannot be eaten.
D. Older memories are far more precious than recent memories and so must be often thought of.

10. The following is an adaptation from Essays by Ralph Waldo Emerson:

All infractions of love and equity in our social relations are speedily punished. They are punished by fear. Whilst I stand in simple relations to my fellow-man, I have no displeasure in meeting him. We meet as water meets water, or as two currents of air mix, with perfect diffusion and interpenetration of nature. But as soon as there is any departure from simplicity, an attempt at halfness, or good for me that is not good for him, my neighbor feels the wrong; he shrinks from me as far as I have shrunk from him; his eyes no longer seek mine; there is war between us; there is hate in him and fear in me.

Based on the passage, what causes the relationship between two people to sour?
A. Simple relations that never progress to deeper understanding of one another.
B. Meeting as two currents of air that must mix together in unity.
C. Any complication of a simple interaction or show of selfishness.
D. A war or argument between mutual friends and neighbors.

  1. B. In the first sentences we see that men without power (farmers) think that power and place are good things while forgetting the cost that “the president” has paid for his position of power in the White House. The rest of the passage builds on this idea, explaining the costs of gaining and maintaining power. This makes answer B correct and answers C and D incorrect. Answer A is incorrect as the author does not state power will always corrupt.
  2. C. This question is answered in the first sentence where the author says that the scholar of the first age received “the world”. He then brooded thereon and gave “it” the new arrangement. From thereon “it” refers to “the world”. All other answers are incorrect readings of the context.
  3. A. In the second sentence the author contends that “society is a… company in which members agree, for the better securing of his bread… to surrender the liberty and culture of the eater.” In other words, in order to survive and have food on the table, people agree to conform to the rules of society. This make answer A correct and the others incorrect.
  4. C. The author says extensively throughout the passage that, even toward strangers, people tend to be “a great deal more kind than is ever spoken”. While Emerson does admit that there is some “selfishness that chills” he maintains that “the whole human family is bathed with an element of love like a fine ether.” This best fits with answer C. Answers A and B are too negative and answer D is incorrect because while some members of society will feel passionate love for some, they do not feel it for all members of society.
  5. D. The author maintains that friendship requires a balance between similarity and difference and holds that no one should “cease… to be himself”. He goes on to say that friends must “recognize the deep identity which beneath these disparities unites them.” In other words, each friend must be true to himself and appreciate the other, making option D correct. Option A is incorrect as the author thinks there should be some differences. Option B is incorrect as there is no discussion of them having to be great. Option C is incorrect as the friends must have a common “deep identity” to unite them.
  6. C. The first sentence tells us that most people think a fortune is necessary for the “competition of this man of the world”. He goes on to say that there are other necessary qualification and that “money is not necessary.” One of the author’s stated qualifications is an affinity which “transcends the habits of clique and caste and makes itself felt by men of all classes.” The author goes on to give examples of men who are not complete me of the world because they lack the ability to transcend class. This makes option C correct and the other options incorrect.
  7. B. The author says that “single lines have a beauty which tempts the ear to pause… yet the sentence is so loaded with meaning, and so linked with its foregoers and followers that the logician is satisfied.” In other words, he sees great beauty in individual lines yet “it is not a merit of lines, but a total merit of the piece.” Both the lines and the whole are beautiful. No other answer encompasses this concept.
  8. A. Answer option B is incorrect as the passage does not talk about appearances. Answer option C is incorrect as prudence has a symbolic character, but is not symbolic in and of itself. Answer option D is incorrect as there is no discussion of seeking good.
  9. A. We see in the passage that older memories are matters of calmest observation, but “not so with our recent actions” because “our affections as yet circulate through it.” Our affections could also be called our emotions. Later on, the author clarifies that “the new deed is yet a part of life”, giving further reason why detached observation is difficult. This makes option A the best answer. The other options are all wrong due to misreading of the passage.
  10. C. In the passage the author describes the simplicity of human interactions, but holds that “as soon as there is any departure from simplicity, an attempts at… good for me that is not good for him, my neighbor feels the wrong” and the hate ensues. Something that is attempted that is good for one’s self but not the neighbor could be described as selfishness, which makes answer C the best option. The author does not describe a relationship that is always simple as bad, making answer A wrong. The simple mixing of two currents of air is seen as the positive relationship prior to it souring, so answer B is wrong, and the war and argument in answer D must happen after the relationship has already soured. It is therefore not causal.

Digital SAT Text Structure and Purpose Reading Practice Questions #2

1. The following is an excerpt from Jane Austin’s Pride and Prejudice. In it, Mrs. Bennet seeks to get to know her new neighbor, Mr. Bingley:

An invitation to dinner was soon afterwards dispatched; and already had Mrs. Bennet planned the courses that were to do credit to her housekeeping, when an answer arrived which deferred it all. Mr. Bingley was obliged to be in town the following day, and consequently unable to accept the honor of their invitation. Mrs. Bennet was quite disconcerted. She could not imagine what business he could have in town so soon after his arrival in Hertfordshire; and she began to fear that he might always be flying about from one place to another, and never settled at Netherfield as he ought to be. What choice best describes the function of the third sentence in the overall structure of the text?

A. It explains Mrs. Bennet’s reaction to the dishes that were planned for dinner that evening.
B. It connects the description of Mr. Bingley’s actions to an explanation of his trip out of town.
C. It shows Mrs. Bennet’s feelings about her invitation to dinner being rejected.
D. It highlights the variety of emotions which Mrs. Bennet is known for.

2. The following is an excerpt from Jane Austin’s Pride and Prejudice:

The village of Longbourn was only one mile from Meryton; a most convenient distance for the young ladies, who were usually tempted thither three or four times a week, to pay their duty to their aunt, and to a milliner’s shop just over the way. The two youngest of the family, Catherine and Lydia, were particularly frequent in these attentions: their minds were more vacant than their sisters’, and when nothing better offered, a walk to Meryton was necessary to amuse their morning hours and furnish conversation for the evening; and, however bare of news the country in general might be, they always contrived to learn some from their aunt. At present, indeed, they were well supplied both with news and happiness by the recent arrival of a militia regiment in the neighborhood; it was to remain the whole winter, and Meryton was the head-quarters.

What choice best describes the function of the underlined portion in the overall structure of the text?
A. It paints a cruel picture of the abilities of Catherine and Lydia that is then qualified by a description of their meaningful trips to Meryton.
B. It addresses one reason why Catherine and Lydia so enjoy their time in Meryton which is then further explained in the subsequent sentences.
C. It shows the author’s general disregard for the shopping, gossip, and visiting that often occupy the time of young women.
D. It shows that the two girls in question have a greater capacity for learning than their sisters, as they haven’t filled their minds with the silly things that are described before and after.

3. The following is an excerpt from Jane Austin’s Pride and Prejudice:

She did at last extort from her father an acknowledgment that the horses were engaged; Jane was therefore obliged to go on horseback, and her mother attended her to the door with many cheerful prognostics of a bad day. Her hopes were answered; Jane had not been gone long before it rained hard. Her sisters were uneasy for her, but her mother was delighted. The rain continued the whole evening without intermission; Jane certainly could not come back.

What choice best describes the function of the underlined portion in the overall structure of the text?
A. It uses a seeming contradiction to foreshadow what many might see as bad luck, but what the mother sees as a very positive turn of events.
B. It shows the mother’s general lack of care for her daughters and the outcome of their day it then transitions into a description of that poor day.
C. It highlights the lack of understanding the mother has as to the weather and her daughter’s lack of safety on horseback.
D. It sets up a paradox which is further built upon in subsequent portions of the passage.

4. The following is an excerpt from Jane Austin’s Pride and Prejudice:

Elizabeth joined them again only to say that her sister was worse, and that she could not leave her. Bingley urged Mr. Jones’s being sent for immediately; while his sisters, convinced that no country advice could be of any service, recommended an express to town for one of the most eminent physicians. This she would not hear of; but she was not so unwilling to comply with their brother’s proposal; and it was settled that Mr. Jones should be sent for early in the morning, if Miss Bennet were not decidedly better. Bingley was quite uncomfortable; his sisters declared that they were miserable. They solaced their wretchedness, however, by duets after supper; while he could find no better relief to his feelings than by giving his housekeeper directions that every possible attention might be paid to the sick lady and her sister.

What choice best describes the function of the final sentence in the overall structure of the text?
A. It completes the description of the dinner party by describing the after-supper entertainment taken up by the guests.
B. It explains the two very different ways that Bingley and his sisters deal with the illness of Miss Bennet.
C. It clarifies the prior description of the search for a good physician for the sick woman.
D. It explains Mr. Bingley’s orders to the housekeeper and her skill in working to heal the young lady who is ill.

5. The following is an excerpt from Jane Austin’s Pride and Prejudice:

As a clergyman, moreover, I feel it my duty to promote and establish the blessing of peace in all families within the reach of my influence; and on these grounds I flatter myself that my present overtures of good-will are highly commendable, and that the circumstance of my being next in the entail of Longbourn estate will be kindly overlooked on your side, and not lead you to reject the offered olive branch. I cannot be otherwise than concerned at being the means of injuring your amiable daughters, and beg leave to apologize for it, as well as to assure you of my readiness to make them every possible amends.

What choice best describes the function of the first sentence in the overall structure of the text?
A. It explains the profession of the speaker so that the reader can empathize with the speaker’s situation.
B. It shows the speaker’s mission to find a way to bring peace to all families which is then followed up with a description of his plan.
C. It explains why the speaker feels it is his duty to make amends by offering up an olive branch.
D.  It introduces the idea of the speaker as a man of authority who each reader should do well to listen to.

6. The following is an excerpt from Jane Austin’s Pride and Prejudice:

In describing to her all the grandeur of Lady Catherine and her mansion, with occasional digressions in praise of his own humble abode, and the improvements it was receiving, he was happily employed until the gentlemen joined them; and he found in Mrs. Philips a very attentive listener, whose opinion of his consequence increased with what she heard, and who was resolving to retail it all among her neighbors as soon as she could. To the girls, who could not listen to their cousin, and who had nothing to do but to wish for an instrument, and examine their own indifferent imitations of china on the mantel-piece, the interval of waiting appeared very long. It was over at last, however. The gentlemen did approach.

What choice best describes the function of the underlined passage in the overall structure of the text?
A. It explains Mrs. Phillips’s plan to sell to her neighbors the china from her mantel-piece.
B. It clarifies why Mrs. Philips was willing to listen to him when his cousins were not.
C. It builds on the previous description of Lady Catherine and her home.
D. It tells the reader what Mrs. Philips is planning to do with the information from the conversation.

7. The following is an excerpt from Jane Austin’s Pride and Prejudice:

Elizabeth had heard, soon after her arrival, that Mr. Darcy was expected there in the course of a few weeks; and though there were not many of her acquaintance whom she did not prefer, his coming would furnish one comparatively new to look at in their Rosings parties, and she might be amused in seeing how hopeless Miss Bingley’s designs on him were, by his behavior to his cousin, for whom he was evidently destined by Lady Catherine, who talked of his coming with the greatest satisfaction, spoke of him in terms of the highest admiration, and seemed almost angry to find that he had already been frequently seen by Miss Lucas and herself.

What choice best describes the function of the underlined portion in the overall structure of the text?
A. To explain Elizabeth’s negative feelings toward Mr. Darcy
B. To clarify why Elizabeth was looking forward to Mr. Darcy’s arrival
C. To introduce Mr. Darcy as one of the many friends of Elizabeth
D. To explain Elizabeth’s reaction toward Mr. Darcy’s arrival

8. The following is an excerpt from Jane Austin’s Pride and Prejudice:

The whole party were in hopes of a letter from Mr. Bennet the next morning, but the post came in without bringing a single line from him. His family knew him to be, on all common occasions, a most negligent and dilatory correspondent; but at such a time they had hoped for exertion. They were forced to conclude, that he had no pleasing intelligence to send; but even of that they would have been glad to be certain.

What choice best describes the function of the underlined phrase in the overall structure of the text?
A. It gives a possible explanation for the situation described in the first sentence.
B. It introduces an aspect of Mr. Bennet’s character that is later disputed.
C. It helps the reader understand why Mr. Bennet lacks pleasing intelligence to send to his family.
D. It shows the family’s depth of knowledge of Mr. Bennet’s character which contrasts with the further characterization of their relationship.

9. The following is an excerpt from Jane Austin’s Pride and Prejudice:

Her husband allowed her to talk on without interruption while the servants remained. But when they had withdrawn, he said to her, “Mrs. Bennet, before you take any, or all of these houses, for your son and daughter, let us come to a right understanding. Into one house in this neighborhood they shall never have admittance. I will not encourage the imprudence of either, by receiving them at Longbourn.”

What choice best describes the function of the final sentence in the overall structure of the text?
A. It explains why all the homes of the neighborhood might be open to them.
B. It clarifies the earlier statement by stating which house the son and daughter will not be allowed into.
C. It completes Mr. Bennet’s explanation of the housing shortage in the neighborhood.
D. It shows Mr. Bennet’s feelings about the imprudence of his wife.

10. The following is an excerpt from Jane Austin’s Pride and Prejudice:

She began now to comprehend that he was exactly the man who, in disposition and talents, would most suit her. His understanding and temper, though unlike her own, would have answered all her wishes. It was a union that must have been to the advantage of both: by her ease and liveliness, his mind might have been softened, his manners improved; and from his judgment, information, and knowledge of the world, she must have received benefit of greater importance.

What choice best describes the function of underlined portion in the overall structure of the text?
A. It explains how each of them might benefit from a relationship with the other.
B. It clarifies the differences between the two and why she is inclined to give him a chance.
C. It explains how he might benefit from a connection to her.
D. It explains how she might benefit from a connection to him.

  1. C. In the first two sentences we learn that Mrs. Bennet had invited Mr. Bingley to dinner and had already planned the meal when Mr. Bingley responded to the invitation with a rejection, for he was to be out of town that evening. This leads to Mrs. Bennet’s reaction of being “disconcerted.” The sentence thereafter gives us further insight into her thoughts on the matter. This makes option C the best answer. Answer A is incorrect as she is disconcerted that her invitation has been refused, not at the dishes that were planned. Answer B is incorrect as the later half of the passage does not describe his trip out of town. Answer D is incorrect for while it does show Mrs. Bennet’s feelings on the matter, it does not show a variety of emotions.
  2. B. The first sentence explains that the young ladies often go to Meryton to visit their aunt and to shop. The next sentence then clarifies that Catherine and Lydia are especially likely to do so because they have vacant minds and needed constant amusement which Meryton provides. This best fits with answer option B. Their vacant minds are one reason why they so enjoy shopping and visiting in Meryton. Answer option A is incorrect as their trips to Meryton are not very meaningful. In addition, while this description seems cruel to our eyes, there is no evidence that the author means it as anything more than an accurate explanation of the characters. Option C is incorrect as there is no evidence that the author is making a general statement about the merit of these activities. Option D is incorrect as the “empty minds” are not shown to be a positive thing.
  3. A. In the underlined portion of the text, the mother is very cheerful about her predictions of a bad day. The following part of the passage clarify that the rain that the mother predicted forces Jane to stay away and “delights” the mother. This best fits with option A. The underlined text seems to be a contradiction, but is setting up the rain, a negative, being seen as a positive in the mother’s eyes. Option B is incorrect as there is no evidence that the mother does not care for her daughter, simply that for some reason she wishes her to be kept away for awhile. Answer C is incorrect as the mother understands the weather quite well and even predicts the rain. Answer D is incorrect as this is not a paradox which is a statement which contradicts itself.
  4. B. The sentence prior to the underlined part tells that Mr. Bingley is uncomfortable and his sisters miserable due to the illness of Miss Bennet. The final sentence describes their reactions to their discomfort and misery. The sisters sing duets to find solace and Mr. Bingley orders his housekeeper to give as much attention to the sick lady and her sister as possible. This best fits answer option B. Answer option A is incorrect as there is no evidence of a dinner party prior to these activities. Answer C is incorrect as it does not have anything to do with a physician (doctor). Answer option D is incorrect as that answer option ignores the sisters’ duets and falsely claims that the housekeeper is working to heal the sick lady when there is no evidence of that.
  5. C. In the underlined portion the speaker explains that he is a clergyman, or a priest. He clarifies that because of this profession he believes he must encourage peace in any family he can influence. In the rest of the passage, then he is apologizing to the listener and offering an olive branch, a well known symbol of peace. This best fits answer C. The speaker, as a clergyman, feels he must apologize and make amends for the sake of peace. Answer A is incorrect as the reader is not asked to empathize with the speaker. Answer B is incorrect as there is no plan to bring peace to all families, rather an attempt to bring peace to one. Answer D is incorrect as the reader is not admonished to listen to the speaker.
  6. D.  In the underlined portion we learn that Mrs. Philips is resolving to retail “it” among her neighbors. The “it” in question is the conversation she is having with the man and “retailing it” could also be described as passing on the information. This makes choice D the best answer. It is not choice A as she is not literally selling anything. It is not choice B, as we do not know why Mrs. Philips is listening to him. It is not choice C as it does not further describe the home in question.
  7. A. Be careful with the negatives in the underlined portion. There are two “nots” which essentially cancel out leaving us with the meaning: there were many of her acquaintances whom she did prefer. In other words, the underlined portion is explaining that Elizabeth does not prefer the company of Mr. Darcy. This best fits with answer option A. The rest of the passage explains why she might look forward to his arrival, but the underlined portion does not, making option B incorrect. Mr. Darcy is a not well like acquaintance of Elizabeth, not a friend, making option C incorrect. Mr. Darcy has not yet arrived, making option D incorrect.
  8. A. In the first sentence we learn that Mr. Bennet had not written them a letter. In the underlined portion this is explained by saying that his family is aware that is a negligent correspondent. In other words, he doesn’t do a good job of writing letters regularly. This makes option A the best answer. Option B is incorrect because the idea of Mr. Bennet being a poor correspondent is not later disputed. Option C is incorrect because it doesn’t not explain why Mr. Bennet has no good news for his family. Option D  is incorrect as there is no later contrast or further description of the relationship.
  9. B. The final statement is clarifying Mr. Bennet’s earlier declaration that there is one house into which the son and daughter will not be allowed: they are not allowed at Longbourn. This makes answer B the best option. It is not answer A as it does not explain why the children are welcome at other homes. Answer C is incorrect as they are not discussing a housing shortage. Answer D is incorrect as Mr. Bennet is discussing the imprudence of the son and daughter, not of his wife.
  10. D. Earlier in the sentence the speaker clarifies that a union between the two would benefit both. Then it explains how he would be benefited (improved manners) and finally, the underlined portion explains the benefit to her (greater importance). This makes option D correct and the others incorrect.

Practice your reading comprehension skills by reading more of Pride and Prejudice for free online:
https://www.gutenberg.org/cache/epub/1342/pg1342-images.html

Digital SAT Standard English Conventions Practice Questions: Punctuation

1.  The hour of noon had just struck, and the few visitors still lingering among the curiosities of the great museum were suddenly startled by the sight of one of the attendants running down the broad, central staircase, loudly shouting, “Close the doors! Let no one out! An accident has ____________________ to leave the building.”

Which choice completes the text so that it conforms to the conventions of Standard English?
A. occurred, while nobody’s
B. occurred, while nobodies
C. occurred, and nobody is
D. occurred, and nobodies

2. She did not answer. She did not even look his way. With a rapid glance into the faces __________________ in one of deep compassion directed toward herself, he repeated his question.

Which choice completes the text so that it conforms to the conventions of Standard English?
A. about him, ending
B. about him; ending
C. about him ending
D. about him ending,

3. With an air of relief Mr. Jewett stepped again into the court and, repelling with hasty gestures the importunities of the small group of men and women who had lacked the courage to follow the more adventurous ______________________ to where the door-man stood on guard over the main entrance.

Which choice completes the text so that it conforms to the conventions of Standard English?
A. ones upstairs—crossed
B. ones upstairs crossed
C. ones upstairs crossed,
D. ones upstairs, crossed

4. They were standing at the foot of the great staircase connecting the two floors. __________________ away on either side, ran the two famous, highly ornamented galleries, with their row of long low arches indicating the five compartments into which they were severally divided.

Which choice completes the text so that it conforms to the conventions of Standard English?
A. Above them, stretching
B. Above, them stretching
C. Above them stretching
D. Above them—stretching

5. The Curator offered his arm. The old man made a move to ___________________ himself up with an air of quiet confidence.

Which choice completes the text so that it conforms to the conventions of Standard English?
A. take it then, drew
B. take it then drew
C. take it—then drew
D. take it: then drew

6. She was near ______________________ being a woman of great nerve, she fought her weakness and waited patiently for the next question. It was different, without doubt, from any she had expected.

Which choice completes the text so that it conforms to the conventions of Standard English?
A. collapsing again, however,
B. collapsing again; however,
C. collapsing again: however,
D. collapsing again, however;

7. As he was meditating how he could best convey to her the necessity of detaining her further, he heard a muttered exclamation from the young woman standing near her and, following the direction of her pointing finger, saw that the strange silence which had fallen upon the room had a _______________ had fainted away in her chair.

Which choice completes the text so that it conforms to the conventions of Standard English?
A. cause; Mrs. Taylor,
B. cause: Mrs. Taylor
C. cause, Mrs. Taylor
D. cause Mrs. Taylor,

8. But the detective was not so hasty. With a thousand things in mind, he stopped to peer along the gallery and down into the court before giving himself away to any prying eye. Satisfied that he might make the desired move with impunity, Mr. Gryce was about to turn in the desired direction when, struck by a ______________ again stopped short.

Which choice completes the text so that it conforms to the conventions of Standard English?
A. new fact: he
B. new fact, he,
C. new fact he
D. new fact, he

9. The detective, thus appealed to, hesitated a ____________________ an irrelevance perhaps natural to the occasion, he inquired where this door so conveniently hidden from the general view led to.

Which choice completes the text so that it conforms to the conventions of Standard English?
A. a moment then; with
B. a moment; then, with
C. a moment, then, with
D. a moment then with

10. The detective, working his way back around the pedestal, cast another glance up and down the _________________ into the court. Still no spying eye, save that of the officer opposite.

Which choice completes the text so that it conforms to the conventions of Standard English?
A. gallery and over
B. gallery, and over
C. gallery and, over
D. gallery: and over

11. Silence. Heads moving, eyes peering, excitement visible in every face, but not a word from anybody. Mr. Gryce turned and pointed up at the clock. ­­­­­­­­­­­­­­­­_________________ still no word from man or woman.

Which choice completest the text so that it conforms to the conventions of Standard English?
A. All looked however
B. All looked: but
C. All looked, or
D. All looked—but

12. He decided upon the northern one, which you will remember was the one holding _________________ __ finding anybody there, no matter whom, would certainly settle the identity of the person responsible for that flying arrow.

Which choice completes the text so that it conforms to the conventions of Standard English?
A. the tapestry; since
B. the tapestry, since,
C. the tapestry since,
D. the tapestry: since,

13. A man was there: man going down—_________________ this man, as he soon saw from his face and uniform, was Correy the attendant.

Which choice completes the text so that it conforms to the conventions of Standard English?
A. no—coming up—
B. no, coming up;
C. no coming up;
D. no, coming up,

14. As he did this, two ____________________ Sweetwater, who had stolen upon the scene, possibly at some intimation from Mr. Gryce, took a step toward them which brought him in alignment with the Englishman, of whose height in comparison with his own he seemed to take careful note; and secondly, the sensitive skin of the foreigner flushed red again as he noticed the Coroner’s sarcastic smile, and heard his dry remark.

Which choice completes the text so that it conforms to the conventions of Standard English?
A. things happened; first
B. things happened; first,
C. things happened: first,
D. things happened: first

15. But nowhere in the great city of which we write on this night of May 23, 1913, was there to be found a scene of greater ______________________ the court and galleries of its famous museum.

Which choice completes the text so that it conforms to the conventions of Standard English?
A. contradictions: than in
B. contradictions than, in
C. contradictions than in
D. contradictions than in,

16. Gems of antique art, casts in which genius had stored its soul and caused to live before us the story of the _________________ from desert sands, friezes from the Parthenon and bas-reliefs from Nineveh and Heliopolis, filled every corner, commanding the eye to satisfy itself in forms of deathless grace or superhuman power.

Which choice completes the text so that it conforms to the conventions of Standard English?
A. ancients pillars
B. ancients, pillars
C. ancients pillars,
D. ancients: pillars

17. The Inspector, finding himself very much disturbed by the doubt just mentioned, felt inclined to _________________ any perceptible advancement had been made by this freak business of his canny subordinate.

Which choice completes the text so that it conforms to the conventions of Standard English?
A. question: whether
B. question whether,
C. question; whether
D. question whether

18. In saying this, Mr. Gryce studiously avoided the _____________________ the Inspector in his turn looked up, then down—anywhere but in the detective’s direction.

Which choice completes the text so that it conforms to the conventions of Standard English?
A. Inspectors’ eye;
B. Inspector’s eye;
C. Inspector’s eye:
D. Inspectors’ eye,

19. For the next three days the impatience of the public met with nothing but disappointment. The police were reticent—more reticent far than __________________________ to add to the facts already published, had little but conjectures to offer.

Which choice completes the text so that it conforms to the conventions of Standard English?
A. usual—and the papers, powerless
B. usual, and the papers, powerless
C. usual—and the papers—powerless
D. usual, and the papers—powerless

20. The other—Carleton Roberts, his bosom friend, and the museum’s chief director—is of a different _____________ no less striking to the eye.

Which choice completes the text so that it conforms to the conventions of Standard English?
A. type: but
B. type, nor
C. type, but
D. type, for

  1. C. “An accident has occurred” is an independent clause. “Nobody is to leave the building” is also an independent clause. Two independent clauses can be connected with a comma and one of the FANBOYS. “While” is not one of the FANBOYS, so A and B are incorrect. “Nobodies” is the plural form of “nobody” so D is incorrect. Answer C is correct because it correctly uses comma FANBOYS and it has the correct format of the word “nobody”.
  2. A. The phrase “ending in one of deep compassion directed toward herself” is extra information in the sentence that is not crucial to the structure of the sentence. Such phrases should be surrounded by commas, dashes, or parentheses. Since there is a comma after the word “herself” there must therefore also be a comma before the word “ending” to correctly bracket the phrase with commas. This makes A the best answer and the other answers incorrect.
  3. D. The phrase starting with the word “repelling” and ending with the word “upstairs” is extra information in the sentence that is not crucial to the structure of the sentence. Such phrases should be surrounded by commas, dashes, or parentheses. Since there is a comma before the word “repelling” there must therefore also be a comma after the word “upstairs” to correctly bracket the phrase with commas. This makes option D the best answer and the other options incorrect.
  4. A. The phrase starting with the word “stretching” and ending with the word “side” is extra information in the sentence that is not crucial to the structure of the sentence. Such phrases should be surrounded by commas, dashes, or parentheses. Since there is a comma after the word “side” there must therefore also be a comma before the word “stretching” to correctly bracket the phrase with commas. This makes option A the best answer and the other options incorrect.
  5. C. The first clause in the second sentence is an independent clause. The second clause is dependent. To connect an independent and dependent clause you should use a comma, however, this is not an option. A dash can replace a comma, giving a heavier pause. This makes answer C the best answer. Answer A puts a comma in the wrong place, answer B does not give punctuation where a pause is needed, answer D is incorrect as the second part of the sentence is not clarifying or explaining the first part.
  6. B. In this situation, the word “however” logically belongs with the second independent clause. The semicolon connecting the two independent clauses, therefore, must go after the word “again”. The word “however” is then an introductory word to the second independent clause, making it logical to place a comma thereafter. This makes option B correct and the other options incorrect.
  7. B. Options C and D are both run on sentences in which two independent clauses are incorrectly connected with a comma. The second clause is clarifying or explaining something from the first clause, namely, the reason the silence has fallen on the room. This makes a colon the best possible answer since a colon connects an independent clause to a clarification or explanation of that clause.
  8. D. The phrase “struck by a new fact” is extra information in the sentence that is not crucial to the structure of the sentence. Such phrases should be surrounded by commas, dashes, or parentheses. Since there is a comma after the word “when” there must therefore also be a comma before the word “again” to correctly bracket the phrase with commas. This makes D the best answer and the other answers incorrect.
  9. B. This sentence is really made up of two sentences. The first one ends after the word “moment”. The second sentence is made up of a dependent clause connected to an independent clause with the comma after the word “occasion”. To connect two sentences, we need to put a period or a semicolon between them. This makes option B the best answer.
  10. A. In this portion of the sentence no clauses are ending or beginning, no extra information exists, and no lists are being enumerated. In other words, there is no reason to put any sort of punctuation. This makes answer A the best option. Answer B would be correct only if the “and” were connecting two independent clauses.
  11. D. In this sentence, there is a contrast between all looking, and no one saying anything. This makes answers A and C incorrect. In addition, the second part of the sentence is not clarifying or explaining the first part, meaning that option B is incorrect. This just leaves answer D which uses the versatile dash to connect two clauses.
  12. A. The first part of this passage is a sentence made up of a dependent clause connected to an independent clause with a comma after the word “one”. The second part of the passage is an independent clause interrupted by the extra information “no matter whom”. To connect these two complete sentences, we need to use a semicolon. This makes option A correct and the other options incorrect.
  13. B. The first part of this passage is a sentence made up of an independent clause with a colon and then clarifying information ending after the word “up.” The second part of the passage is an independent clause interrupted by the extra information “as he soon saw from his face and uniform.” In order to connect these two sentences, we need to use a semicolon. This eliminates option D. The confusion of having two dashes around essential information eliminates option A. The need for a pause after “no” leads us to B as the best answer.
  14. C. A colon goes after an independent clause and before a clarification or explanation of that independent clause. A comma goes after an introductory word or phrase like “first”. This makes answer option C the best choice.
  15. C. In this portion of the sentence no clauses are ending or beginning, no extra information exists, and no lists are being enumerated. In other words, there is no reason to put any sort of punctuation.
  16. B. This sentence starts with a list. Among the items in the list are “casts in which genius had stored its soul and caused to live before us the story of the ancients” and “pillars from desert sands”. These two items must be separated with a comma, making option B the correct answer.
  17. D. In this portion of the sentence no clauses are ending or beginning, no extra information exists, and no lists are being enumerated. In other words, there is no reason to put any sort of punctuation.
  18. B. In this sentence there is a single inspector who has possession over his eye. This means that the apostrophe must go before the “s”, making options A and D incorrect. In addition, we are connecting two independent clauses, making the semicolon the best option.
  19. A. “More reticent far than usual” is extra information in the sentence. Since it begins with a dash before the word “more” we must end it with a dash after the word “usual”. The phrase “powerless to add to the facts already published” is also extra information. Since it ends with a comma after the word “published” we must start it with a comma before the word “powerless”. This makes option A the correct answer.
  20. C. The first clause of the sentence says that Carleton Roberts is a different type, this contrasts with him being “no less striking”. The contrasting word “but” therefore fits the author’s meaning. A comma to connect the independent clause to a dependent one would also be appropriate, making option C the best answer.

All passages are adapted or taken from The Mystery of the Hasty Arrow by Anna Katharine Green. You can read who whole exciting mystery novel about a murder in a museum for free online:

https://www.gutenberg.org/cache/epub/17763/pg17763-images.html

Digital SAT Practice: Writing/Verb Choice Questions

1. The officer nodded, took the boy by the arm, and in a trice ___________ with him into the adjoining store.

Which choice completes the text so that it conforms to the conventions of standard English?
A. have disappeared
B. disappears
C. disappeared
D. disappear

2. ________ the white light burn on, Mr. Gryce, by a characteristic effort, shifted his attention to the walls, covered, as I have said, with tapestries and curios.

Which choice completes the text so that it conforms to the conventions of standard English?
A. Letting
B. Let
C. Had let
D. Have let

3. And sure enough, in another instant this strange being, losing all semblance to his former self, entered upon a series of pantomimic actions which to the two men who watched him seemed both to explain and illustrate the crime which _________________________ there.

Which choice completes the text so that it conforms to the conventions of standard English?
A. had just been enacted
B. have just been enacted
C. were just enacted
D. had just enacted

4. The butler’s lips opened and a string of strange gutturals poured forth, while with his one disengaged hand (for the other was held to his side by Styles) he _________ his ears and his lips, and violently shook his head.

Which choice completes the text so that it conforms to the conventions of standard English?
A. touches
B. was touching
C. touched
D. had touched

5. This absence of the usual means of eliciting knowledge from the surrounding people, adds to, rather than detracts from, the interest which Mr. Gryce  feels in the case, and a little before midnight the army of reporters, medical men, officials, and such others as had followed in the coroner’s wake, _______ out of the front door and leave him again, for a few hours at least, master of the situation.

Which choice completes the text so that it conforms to the conventions of standard English?
A. files
B. file
C. filing
D. filed

6. The hour was late, and only certain portions of the city showed any real activity. Into one of these thoroughfares they presently came, and before the darkened window of one of the lesser shops ________, while Jake pointed out the two stuffed frogs engaged with miniature swords in mortal combat at which he had been looking when the lady came up and spoke to him.

Which choice completes the text so that it conforms to the conventions of standard English?
A. paused,
B. pausing,
C. pause,
D. pauses,

7. The officer went out, and Mr. Gryce sat for a few moments communing with himself, during which he took out a little package from his pocket, and __________ out on his desk the five little spangles it contained, regarded them intently.

Which choice completes the text so that it conforms to the conventions of standard English?
A. emptied
B. empties
C. had emptied
D. emptying

8. Sweetwater, to whom the song of the sirens would have sounded less sweet, listened with delight and ______________ with a frank smile.

Which choice completes the text so that it conforms to the conventions of standard English?
A. respond
B. had responded
C. responds
D responded

9. This time he approached with considerable feebleness, passed slowly into the study, ­­­­­­­­­­­______________ to the table, and reached out his hands as if to lift something which he expected to find there.

Which choice completes the text so that it conforms to the conventions of standard English?
A. advances
B. advanced
C. have advanced
D. advance

10. But her ears, and attention, _______________toward two girls chatting on a bench near her as freely as if they were quite alone on the lawn.

Which choice completes the text so that it conforms to the conventions of standard English?
A. was turned
B. are turned
C. were turned
D. is turned

11. Was it that courage comes with despair? Or was he too absorbed in his own misery to note the shadow it cast about him? His brooding brow and vacant eye _______ of a mind withdrawn from present surroundings.

Which choice completes the text so that it conforms to the conventions of standard English?
A. speaks
B. speak
C. spoke
D. spoken

12. So I wrote to my brother, Felix Cadwalader, or, rather, Felix Adams, as he preferred to be called in later years for family reasons entirely disconnected with the matter of his sudden demise, and, ________ him I had become interested in a young girl of good family and some wealth, asked him to settle upon me a certain sum which would enable me to marry her with some feeling of self-respect.

Which choice completes the text so that it conforms to the conventions of standard English?
A. told
B. tells
C. telling
D. had told

13. Eva, to whom I had said little of this brother, certainly nothing which would lead her to anticipate ___________ either so handsome a man or one of such mental poise and imposing character, looked frightened and a trifle awe-struck.

Which choice completes the text so that it conforms to the conventions of standard English?
A. seeing
B. to see
C. having seen
D. saw

14. Your stay in Mr. Adams’s house was quite productive, ma’am. Did you prolong it after the departure of this old man?” “No, sir, I _____________my fill of the mysterious, and left immediately after him.

Which choice completes the text so that it conforms to the conventions of standard English?
A. had had
B. have had
C. was having
D. could have

15. Mr. Gryce, with something of the instinct and much of the deftness of a housewife, proceeded to pull up a couple of rugs from the parlor floor and ­­_________ them over these openings.

Which choice completes the text so that it conforms to the conventions of standard English?
A. strung
B. string
C. had strung
D. strings

16. Miss Butterworth drew a long breath, ________ Mr. Gryce with some curiosity, and then triumphantly exclaimed, “Can you read the meaning of all that? I think I can.”

Which choice completes the text so that it conforms to the conventions of standard English?
A. eyes
B. had eyed
C. eying
D. eyed

17. Mr. Gryce, whose eye is travelling over the wall, reaches over her shoulder to one of the dozen pictures hanging at intervals from the bottom to the top of the staircase, and pulling it away from the wall, on which it hangs decidedly askew, _________ a round opening through which pours a ray of blue light which can only proceed from the vault of the adjoining study.

Which choice completes the text so that it conforms to the conventions of standard English?
A. revealed
B. revealing
C. reveals
D. has revealed

18. The study—that most remarkable of rooms—________ a secret which has not been imparted to you; a very peculiar one, madam, which was revealed to me in a rather startling manner.

Which choice completes the text so that it conforms to the conventions of standard English?

A. did contain
B. contains
C. is containing
D. contain

19. Young Sweetwater, who was now all nerve, enthusiasm, and hope, ______________.

Which choice completes the text so that it conforms to the conventions of standard English?
A. bow
B. bowed
C. bowing
D. have bowed

20. The two gentlemen, on the contrary, with an air of total indifference to her proximity, continued their walk until they reached the end of the piazza, and then __________ and proceeded mechanically to retrace their steps.

Which choice completes the text so that it conforms to the conventions of standard English?
A. turned
B. turning
C. turn
D. turns

  1. C. In the sentence, the officer does three things. He nods, takes the boys arm, and disappears. In the sentence the first two actions are in the past tense “nodded, took”. In order to maintain parallel structure, the third verb should also be in the past tense. This makes option C the best answer. Options B and D are in the present, and option A is the plural present perfect, not the singular simple past.
  2. A. In English we can use the present participle or gerund (word ending in ing) to indicate that something is currently happening at the point in time being discussed. In this case, Mr. Gryce is letting the light burn while in the past he shifted attention to the walls. The other answers do not appropriately choose a tense that allows the reader to understand this.
  3. A. Answer options C and D are incorrect because they say that the crime enacted something instead of the crime being enacted itself. Option B is incorrect because it uses the plural “have” to describe the singular “crime”. Answer option A is therefore the best choice because it uses the singular “had” and correctly explains that it is the crime that was enacted.
  4. C. In this sentence, the butler is doing three things. His lips open, he touches his ears and lips, and he shakes his head. All three of these actions must be in the same tense. This makes option C correct as it matches “touched” to “opened” and “shook”.
  5. A. Be careful in identifying the subject of your verb in such lengthy sentences. Who is it who is filing out the front door? It isn’t Mr. Gryce. Nor is it the reporters, medical men, officials etc. (the subject of your verb will never be in a prepositional phrase).  Rather, the subject of the verb is “army”. Army is a singular noun, so you much choose the singular “files”. In addition, the passage is in present tense, making option D incorrect.
  6. A.  Since the passage is in the past tense and “paused” is in the past tense, answer A is the only correct answer.
  7. D. While the passage is in the past tense, at that point in the past, Mr. Gryce is presently “emptying” the package out onto his desk. One way to be clued into this is to put the answers into the context of the last part of the passage “regarded them intently.” Which only makes sense when answer D is selected. If answer A is selected the sentence is awkward.
  8. D. Sweetwater is doing two things—match your answer to “listened” and the correct answer “responded” becomes obvious.
  9. B. The man did four things: approached, passed, advanced, and reached. Only answer option B correctly matches “advanced” with the others.
  10. C. Since the subject “ears” is plural, options A and D are incorrect as they have singular verbs “was” and “is”. Since the sentence is in the past tense, the verb “were” is the most appropriate answer.
  11. C. This passage is in the simple past, so our verb must be “spoke”. Options A and B are in the present, and option D is the past participle which would go with a helping verb to create the past perfect instead of building the simple past.
  12. C. Even though this passage is in the past tense, this particular sentence is put into the past with the words “had become”. What the author is saying is that at a point in the past, the author of the letter was currently “telling” his brother that he (the writer) had become interested. The only answer that fits this complicated idea is answer option C.
  13. A. Since Eva is anticipating something, that something will be happening in the future. This means that we must use the gerund form—that is—the verb with “ing” at the end. This makes option A the best answer and the other options incorrect.
  14. A. In the past, the lady had already had her fill of the mysterious. She “had had” it. While this seems intuitively wrong, it is the correct way to express that in the past someone already had something. Option B would be used if the subject were plural instead of a singular person.
  15. B. Mr. Gryce does two things the first is to “pull” and the second, therefore must be to “string” so that the tenses of the verbs match.
  16. D. Miss Butterworth “drew”, “eyed”, and “exclaimed”. In order for her three actions to be in parallel structure (all in the simple past), we must pick option D.
  17. C. Since this passage is describing Mr. Gryce’s actions in the present tense, the best option is option C, the third person present singular. Options A and D are forms of the past tense and option B is the gerund form.
  18. B. The subject of this verb is singular: the study. This make option be the only correct answer since it is the third person singular conjugation of the verb “contain”.
  19. B. The subject of the verb is “Young Sweetwater”. When you put the subject directly in front of the answers it becomes clear that only option B is the right verb. Answers A and D are both plural while the subject is singular. Option C is the gerund form which is not appropriate in this context.
  20. A. Make sure that your answer is parallel in form to the second verb “proceeded”. In this case, that would mean choosing answer option A: turned.  The other options are not parallel.


All passages have been adapted from The Circular Study by Anna Katharine Green. Read more of this mystery novel on project Gutenberg:

https://www.gutenberg.org/cache/epub/18761/pg18761-images.html


Digital SAT Poetry Practice Reading Questions #4

1. The following is a poem by Alfred Tenison:

When cats run home and light is come,
And dew is cold upon the ground,
And the far-off stream is dumb,
And the whirring sail goes round,
And the whirring sail goes round;
Alone and warming his five wits,
The white owl in the belfry sits.

When merry milkmaids click the latch,
And rarely smells the new-mown hay,
And the cock hath sung beneath the thatch
Twice or thrice his roundelay,
Twice or thrice his roundelay;
Alone and warming his five wits,
The white owl in the belfry sits.

What is the main idea of the text?
A. Cats and milkmaids share much in common: both are seen by the owl.
B. As the sun rises and the world comes to life, the owl settles in to rest.
C. The routine of each day is predictable and common.
D. The cat and the rooster are the waking signs of each morning.

2. “An Incident of the French Camp” is a poem by Robert Browning. Having read it, a student claims that the town of Ratisbon has been taken. Which quotation from the text best supports the claim?

A. You know, we French storm’d Ratisbon/ A mile or so away/ On a little mound, Napoleon/ Stood on our storming-day;
B. Just as perhaps he mus’d “My plans/ That soar, to earth may fall. Let once my army leader Lannes/ Waver at yonder wall,”
C. “Well,” cried he, “Emperor, by God’s grace/ We’ve got you Ratisbon!/ The Marshal’s in the market-place/ And you’ll be there anon!”
D. “You’re wounded!” “Nay,” the soldier’s pride/ Touched to the quick, he said: “I’m killed, Sire!” And his chief beside, Smiling the boy fell dead.

3. The following is the poem “Old Ironsides” by Oliver Wendell Homes:

Ay, tear her tattered ensign down!
Long has it waved on high,
And many an eye has danced to see
That banner in the sky;
Beneath it rung the battle shout,
And burst the cannon’s roar;—
The meteor of the ocean air
Shall sweep the clouds no more.

Her deck, once red with heroes’ blood,
Where knelt the vanquished foe,
When winds were hurrying o’er the flood
And waves were white below.
No more shall feel the victor’s tread,
Or know the conquered knee;
The harpies of the shore shall pluck
The eagle of the sea!

O, better that her shattered hulk
Should sink beneath the wave;
Her thunders shook the mighty deep,
And there should be her grave;
Nail to the mast her holy flag,
Set every threadbare sail,
And give her to the god of storms,
The lightning and the gale!

Based on the text, what fate would Holmes prefer for Old Ironsides?
A. That she be destroyed by a lightning strike
B. That she be taken ashore and preserved for posterity
C. That she lose her flag and decks
D. That she sink in glory to the depths of the sea

4. “Warren’s Address to the American Soldiers” is a poem by John Peirpont. A writer, knowing that that the poem echoes Warren’s rallying cry to American soldiers, claims that it aims to glorify a coming battle. Which quotation from the poem most effectively illustrates the claim?

A. Look behind you! They’re afire!/ And, before you, see/ Who have done it! – From the vale/ On they come!
B. Will ye look for greener graves?/ Hope ye mercy still?
C. What’s the mercy despots feel?/ Hear it in that battle-peal!
D. In the God of battles trust!/ Die we may, and die we must/ But, O, where can dust to dust/ Be consigned so well.

5. “My Own Shall Come to Me” is a poem by John Burroughs. A teacher tells her class that it is a poem about patience and the inevitability of fate. Which quotation from “My Own Shall Come to Me” most effectively illustrates the claim?

A. I rave no more ‘gainst time or fate/For lo! My own shall come to me…/ No wind can drive my bark astray,/ Nor change the tide of destiny.
B. The stars come nightly to the sky;/ the tidal wave comes to the sea;/ Nor time, nor space, nor deep, nor high,/ Can keep my own away from me.
C. The waters know their own and draw/ The brook that springs in yonder heights;/ So flows the good with equal law/ Unto the soul of pure delights.
D. Serene I fold my hands and wait,/ Nor care for wind, nor tide, nor sea…/ What matter if I stand alone?/ I wait with joy the coming years;

6. The following is the poem “A Wish” by Samuel Rogers:
 
Mine be a cot beside the hill;
A bee-hive’s hum shall soothe my ear;
A willowy brook that turns a mill
With many a fall shall linger near.

The swallow, oft, beneath my thatch
Shall twitter from her clay-built nest;
Oft shall the pilgrim lift the latch,
And share my meal, a welcome guest.

Around my ivied porch shall spring
Each fragrant flower that drinks the dew;
And Lucy, at her wheel, shall sing
In russet gown and apron blue.

The village church among the trees,
Where first our marriage-vows were given,
With merry peals shall swell the breeze
And point with taper spire to Heaven.

Which choice best describes the function of the underlined portion in the text as a whole?
A. To describe the garden of the narrator
B. To paint a picture of an idyllic country life
C. To help the reader visualize all that the narrator has lost
D. To illustrate the domestic part of what the narrator is wishing for

7. “L’Envoi” is a poem by Rudyard Kipling. A commenter claims that if the entire poem is read as if Kipling approving of what he describes, then Kipling believes that artists should paint not because they must, but for the love of it. Which quotation from “L’Envoi” most effectively illustrates the claim?

A. And only the Master shall praise us/ and only the Master shall blame;/ And no one shall work for money,/ and no one shall work for fame;/ But each for the joy of the working
B. And those who were good shall be happy: they shall sit in a golden chair;/ They shall splash at a ten-league canvas with brushes of comet’s hair;
C. When the oldest colors have faded, and the youngest critic has died,/ We shall rest, and, faith, we shall need it—lie down for an eon or two.
D. They shall find real saints to draw from – Magdalene, Peter, and Paul;/ They shall work for an age at a sitting and never be tired at all!

8. The following is the poem “Little Things” by Ebenezer Cobham Brewer:

Little drops of water,
Little grains of sand,
Make the mighty ocean
And the pleasant land.

Thus the little minutes,
Humble though they be,
Make the mighty ages
Of eternity.

8. Which choice best states the main purpose of the text?
A. To illustrate how small and individual moments build into all of history.
B. To discuss time lost while visiting the ocean.
C. To contrast the relentless progression of time with the endless movement of the ocean
D. To encourage the reader to take their time as they go through life.

9. The following is the poem “The Butterfly and the Bee” by William Lisle: BowlesMethought I heard a butterfly
Say to a laboring bee:
“Thou hast no colors of the sky
On painted wings like me.”

“Poor child of vanity! those dyes,
And colors bright and rare,”
With mild reproof, the bee replies,
“Are all beneath my care.

“Content I toil from morn to eve,
And scorning idleness,
To tribes of gaudy sloth I leave
The vanity of dress.”

What is the main idea of the text?
A. In an imaginary conversation, a butterfly and a bee have little in common.
B. As a personal quality, work ethic is far preferable to beauty.
C. Being judgmental of others is not an admirable pastime.
D. Even creatures as small as bees have lessons to teach us all.

10. The following is the poem “Ingratitude” by William Shakespeare: Blow, blow, thou winter wind,

Thou are not so unkind
As man’s ingratitude;
Thy tooth is not so keen
Because thou are not seen,
Although thy breath be rude.

Freeze, freeze, thou bitter sky,
Thou dost not bite so nigh
As benefits forgot;
Though thou the waters warp,
Thy sting is not so sharp
As friend remembered not.

Which choice best states the main purpose of the text?
A. To explain that winter is a harsh time through which all men must suffer.
B. To illustrate that emotional pain can hurt far worse than physical pain.
C. To paint a picture of various harms that many befall the narrator.
D. To show the readers the dangers of making friends with the wrong people.


Answer Explanations

  1. B. Each stanza starts with a description of the morning. These descriptions include the cat coming home, the light coming, the milkmaid leaving her house, and the rooster crowing. Each stanza ends with the owl sitting and warming himself in his belfry. This best fits answer option B. We have no evidence that the owl sees the world waking, making option A incorrect. The daily routine is described, but not focused on as predictable, making option C incorrect. The cat and the rooster are only mentioned to highlight the time of day, making option D incorrect.
  2. C. Choice C best shows that Ratisbon has been taken. In it, we hear words spoken to the Emperor that “we’ve got you Ratisbon”. The army has taken the city and the Emperor will be in Ratisbon’s market place “anon” or soon. Option A proves that an army has stormed the town, but not that the town fell to that army. Option B shows that the leader is concerned they might not succeed in their military efforts. Option D shows the death of the messenger who came to the emperor.
  3. D. When reading the second stanza, we see that Holmes would prefer that the “harpies of the shore” not pluck “the eagle of the sea”. He says “O, better that her shattered hulk should sink beneath the wave.” Through the rest of the second stanza he clarifies that he would prefer to give her to the “god of storms”. He does not say that he wants her to be struck by lightning, so answer A is too specific. Answer B is the opposite of his preference. Option C is not described in the poem.
  4. D. Answer option D best glorifies the coming battle. It calls on the soldiers to trust in the God of battles, tells them that some will die, and then explains that such a death (dust to dust) will be consigned well. Answer option A talks of an oncoming foe, but does not glorify. Option B asks the soldiers if they are hoping to avoid the battle. Option C points out that the opposition will have no mercy.
  5. A. Option A best shows both patience and fate. The narrator does not rave against time or fate, but rather waits patiently for his “own” to come to him. He knows he cannot change destiny, so he does not try. Option B could prove his feelings about fate, but they do not show his patience. Option C speaks to neither fate nor patience. Option D speaks to patience, but not fate.
  6. D. The key in answering this question is in the title of the poem: A Wish. The entire poem describes what the narrator wishes for. Each stanza describes part of his wish. The underlined stanza describes his home and wife—these are domestic things, making option D the best answer. Answer A is incorrect as the garden is not the only thing described in the stanza and we have no evidence that the wish has been granted and that the garden actually exists. Option B is incorrect as this is not a generic picture of country life, but a specific hope for the future that the narrator has. Option C is incorrect as there is no evidence that the narrator once had this and lost it.
  7. A. A key note in the question is that the assumption is that Kipling approves of what he describes in the poem. This makes answer A the best option, as Kipling would approve of artists working, not for money or fame, but rather because it brings them joy. Option B describes happy painters, but not their motivations. Option C describes exhausted artists taking a rest. Option D describes what they paint.
  8. A. The first stanza describes how the vastness of land and sea are made up of tiny grains of sand and drops of water. This illustration, then is used in the second stanza to show how tiny moments of time make up all of extant history. This best fits answer option A. Answer option B is too literal. Answer options C and D draw conclusions not supported by the text.
  9. B. In the text, the butterfly points out that the bee does not have beautiful wings. The bee then pities and reproofs the butterfly for her vanity and highlights the hardworking nature of the humble bee. This best fits with option B. Answer option A is incorrect as the main idea is that of vanity and work, not that bees and butterflies have nothing in common. Answer option C is incorrect as while it is true, it is not the main idea. Option D is true, but not the message of the author.
  10. B. In each of the two stanzas, Shakespeare first describes horrible winter conditions and then says that ingratitude and “friend remembered not” respectively are far worse. These emotional pains are pointed out as worse than freezing winter weather, making option B the best choice. It isn’t option A as there is an emotional component to the poem. It isn’t option C as Shakespeare doesn’t mean to highlight physical harms, rather emotional ones. It isn’t D as there is no evidence that the emotional pain comes only from “the wrong people”.


Each poem is taken from “Poems Every Child Should Know”, edited by Mary E. Burt. If you wish to improve your poetry skills, you can read more poems from this book on Project Gutenberg:

https://www.gutenberg.org/cache/epub/16436/pg16436-images.html



Digital SAT Poetry Practice Reading Questions #3

1. “In a Library” is a poem written by Emily Dickinson. A student who read the poem claims that Dickinson personifies an antique book, seeing it as a man whom she joys to meet. Which quotation from “In a Library” most effectively illustrates the claim?

A. What interested scholars most/ what competitions ran/ When Plato was a certainty/ And Sophocles a man
B. He traverses familiar/ As one should come to town/ And tell you all your dreams were true/ He lived where dreams were sown.
C. His presence is enchantment/ you beg him not to go/ old volumes shake their vellum heads/ and tantalize just so.
D. A precious moldering pleasure ‘tis/ To meet an antique book.

2. The following is the poem The Book of Martyrs (adapted), by Emily Dickinson

Read, sweet, how others strove,
Till we are stouter;
What they renounced,
Till we are less afraid;
How many times they bore
The faithful witness,
Till we are helped,
As if a kingdom cared!

Read then of faith
That shone above the fray;
Clear strains of hymn
The river could not drown;
Brave names of men
And celestial women,
Passed out of record
Into renown!

What is the main idea of the text?

A. Students should read diligently and memorize names of men and women who have passed away.
B. We should read of and look to the examples of those who went before us to gain strength, courage, and help.
C. Those who came before us are gone, passed out of the record, and no longer worth the effort to learn about.
D. Life is difficult; those older than us can attest to the fear and weakness of this mortal life.

3. The following is a poem by Emily Dickinson:

If you were coming in the fall,
I’d brush the summer by
With half a smile and half a spurn,
As housewives do a fly.

If I could see you in a year,
I’d wind the months in balls,
And put them each in separate drawers,
Until their time befalls.

If only centuries delayed,
I’d count them on my hand,
Subtracting till my fingers dropped
Into Van Diemen’s land.

If certain, when this life was out,
That yours and mine should be,
I’d toss it yonder like a rind,
And taste eternity. But now, all ignorant of the length
Of time’s uncertain wing,
It goads me, like the goblin bee,
That will not state its sting.

Based on the text, what is it that goads the narrator “like the goblin bee”?

A. The constant turn of the seasons of the year
B. The centuries of life she has lived
C. A fly, buzzing around her head
D. The uncertainly of time

4. “In Vain” is a poem by Emily Dickinson. A literature professor tells her class that the poem shows Dickinson’s anxiety around losing loved ones. Which quotation from the poem best illustrates this claim?

A. I cannot live with you/ It would be life/ And life is over there/ behind the shelf.
B. Glow plain and foreign/ On my homesick eye/ Except that you, than he/ Shone closer by.
C. And were you lost, I would be/ Though my name/ Rang loudest/ On the heavenly frame.
D. So we must keep apart/ you there, I here/ With just the door ajar

­

5. The following is a poem by Emily Dickinson:

The pedigree of honey
Does not concern the bee;
A clover, any time, to him
Is aristocracy.

Which choice best states the main purpose of the text?

A. To explain what is most important to the common honey bee.
B. To make a statement on the aristocratic nature of pollinating insects.
C. To make a point about hierarchy, using a nature metaphor.
D. To show the reader what is truly important in life.

6. “A Service of Song” is a poem by Emily Dickinson, a poet who was active in 19th century America.
Note: a bobolink is a type of songbird.

Some keep the Sabbath going to church;
I keep it staying at home,
With a bobolink for a chorister,
And an orchard for a dome.

Some keep the Sabbath in surplice;
I just wear my wings,
And instead of tolling the bell for church,
Our little sexton sings.

God preaches, — a noted clergyman, —
And the sermon is never long;
So instead of getting to heaven at last,
I’m going all along!

Which choice best describes the function of the underlined portion in the text as a whole?

A. It paints a picture of an unconventional practice which is later clarified.
B. It shows that the narrator is a rebel, a fact that has later consequences.
C. It helps the reader understand why the narrator chooses to stay home.
D. It singles out the narrator as a chorister and bird lover.

7. A student reads the poem “The Grass” by Emily Dickinson and claims that the narrator envies the simple life of grass. Which quotation from “The Grass” most effectively illustrates the claim?

A. The grass so little has to do/A sphere of simple green/ With only butterflies to brood/And bees to entertain.
B. The grass so little has to do/ I wish I were the hay!
C. And even when it dies, to pass/ in odors so divine
D. And stir all day to pretty tunes/ The breezes fetch along/ And hold the sunshine in its lap/ and bow to everything.

8. The following is a poem by Emily Dickinson.

I died for beauty, but was scarce
Adjusted in the tomb,
When one who died for truth was lain
In an adjoining room.

He questioned softly why I failed?
“For beauty,” I replied.
“And I for truth, — the two are one;
We brethren are,” he said.

And so, as kinsmen met a night,
We talked between the rooms,
Until the moss had reached our lips,
And covered up our names. Which choice best states the main idea of the text?

A. Truth is beauty and beauty is truth.
B. Finding like-minded people is worth any effort.
C. There is beauty and truth in the death of evil people.
D. Once deceased, people are as quickly forgotten as moss covers their gravestones.

9. A student reads an untitled poem by Emily Dickinson and claims that the man discussed in the passage is dying or dead. Which quotation from the poem most effectively illustrates the claim?

A. What was his furthest mind, of home, or God,/ Or what the distant say/At news that he ceased human nature/On such a day?
B. To know just how he suffered would be dear/ To know if any human eyes were near
C. And wishes, had he any?/ Just his sigh, accented,/ Had been legible to me./ And was he confident until
D. Was he afraid, or tranquil?/ Might he know/ How conscious consciousness could grow,/ Till love that was, and love too blest to be,/Meet —

10. “Trying to Forget” is a poem by Emily Dickinson:

Bereaved of all, I went abroad,
  No less bereaved to be
Upon a new peninsula, —
  The grave preceded me,

Obtained my lodgings ere myself,
  And when I sought my bed,
The grave it was, reposed upon
   The pillow for my head.

I waked, to find it first awake,
   I rose, — it followed me;
I tried to drop it in the crowd,
   To lose it in the sea, In cups of artificial drowse
  To sleep its shape away, —
The grave was finished, but the spade
  Remained in memory.

Which choice best states the main purpose of the text?

A. To paint a realistic picture of the grief of a traveler.
B. To metaphorically explain how the loss of a loved one effects the narrator.
C. To describe a troubled vacation.
D. To illustrate the narrator’s struggle with the thought of death.


Answer Explanations

  1. C. Answer option A gives no evidence that a book is being personified. Rather, it says that Sophocles (a person) was a man. Answer option B would be good evidence if we knew that the author was talking about a book. However, it could just as likely be about an actual man. Answer option D shows the author’s joy in “meeting” a book, but does not go so far as to personify the book as a man. Option C, therefore is the best option. It refers to the “old volumes” as “him” and it says that his presence is “enchantment”. Meaning, the author gets joy from the presence.
  2. B. The first stanza of the text commands us to read about how others strove, renounced, and bore witness until we are stouter, less afraid, and helped. The second stanza commands us to read of the faith that others had and the men and women who passed into renown. Passing into renown could be otherwise stated as passing from this life into memory. The poems main idea, therefore best fits with option B. Option A is incorrect as there is no encouragement of memorization. Option C is the opposite of the message of the poem. Option D is incorrect as the author wants us to look to those gone as examples of good things, not fear and weakness.
  3. D. In the first stanza of the poem, the author discusses what she would do if “you” were coming in the fall. She is waiting for seasons to pass. In the second stanza she talks about years. In the third stanza she is waiting centuries. However, in the last stanza she talks about how she doesn’t know which of these (seasons, years, centuries) she will have to wait. She is “ignorant of the length of time’s uncertain wing”. Time, therefore is the bee that goads her. D is the only answer that makes sense.   
  4. C. Answer option C tells of the narrator’s belief that even heavenly exultation would not be enough for her if “you were lost”. This shows her intense fear around losing a loved one. She believes she would be lost no matter what, were the object of the poem also lost. Answer option A is incorrect as it shows that Dickinson does not want to live with the object of the poem. Answer option B is incorrect as it shows her attachment to the object of the poem, not her anxiety about losing him or her. Answer option D is incorrect as it shows that the narrator wishes to remain apart from the object of the poem, not that she fears losing him or her.
  5. C. The poem is saying that bees like all honey regardless of pedigree and that clovers are all seen as “aristocracy”. The metaphor applies to humans, that we should all regard one another as bees regard honey and aristocracy. This makes C the best answer.  Answer A is incorrect as we find out that bees regard all honey and clover equally, not with some more important than others. Answer B is incorrect, as the author is not saying that bees are aristocratic. Answer option D is incorrect, as the poem makes not allusion to what humans should value the most, rather, that we should value all humans equally. 
  6. A. The underlined portion points out that the narrator, instead of going to church on Sunday, which would have been the accepted practice in 19th century America, stays at home. The songbirds are her choir and the orchard is the roof of her church. This is an unconventional practice for the time. Later in the poem, she clarifies that she does this as she believes that this place, the orchard, is her heaven where she has been “going all along”. This makes option A the best answer and the other options incorrect.
  7. B. Answer option B shows the envy of the narrator as she wishes she were the hay. Hay is just another word for grass. The other options all show what might be enviable qualities of grass, but not that the narrator actually envies it.
  8. A. The poem tells a story of someone who dies and, “in the tomb” meets a man who asks why she died. She replies that she died for beauty and he responds by saying that he died for truth and that “the two are one; we brethren are”. They then form a bond, a friendship, in death, over their similar causes. This best fits with answer option A. Answer B is incorrect as the main idea is that truth and beauty are the same and worth dying for, not that finding like-minded people is worth dying for. Answer C is incorrect as there is no evidence either of these people are evil. Answer option D is incorrect, as it captures only a part of the last stanza, not the main idea of the whole text.
  9. A. Answer option A is the best option because it talks about the man as having “ceased human nature” which is another way of saying to die. This excerpt is asking what the man was thinking of as he passed away. The other options do not as clearly prove the claim, but rather offer only peripheral evidence in the context of answer option A.
  10. D. In the first stanza the author goes to “a new peninsula” but the grave proceeds her there. In the second stanza she finds a place to stay and yet the grave waited for her in her new bed. In the third stanza, the grave follows her as she wakes and goes about her day. She can only escape these thoughts of grave and death when she sleeps in the fourth stanza. This makes option D the best answer.


All poems were taken from Book of Poems by Emily Dickinson.  You can read more of this book here:
https://www.gutenberg.org/cache/epub/12242/pg12242-images.html

Digital SAT Poetry Practice Reading Questions #2

1. The following is “The Land of Counterpane” by Robert Louis Stevenson:

When I was sick and lay a-bed,
I had two pillows at my head,
And all my toys beside me lay
To keep me happy all the day.

And sometimes for an hour or so
I watched my leaden soldiers go,
With different uniforms and drills,
Among the bed-clothes, through the hills;

And sometimes sent my ships in fleets
All up and down among the sheets;
Or brought my trees and houses out,
And planted cities all about.

I was the giant great and still
That sits upon the pillow-hill,
And sees before him, dale and plain,
The pleasant land of counterpane.

Which of the following options best explains the main purpose of the poem “The Land of Counterpane” by Robert Louis Stevenson?

A. To illustrate a child’s fertile imagination.
B. To tell a story of high adventure.
C. To show the journeys of a young child.
D. To build an understanding of physical malady.

2. The following is the poem “The Sun Travels” By Robert Louis Stevenson:

The sun is not a-bed, when I
At night upon my pillow lie;
Still round the earth his way he takes,
And morning after morning makes.

While here at home, in shining day,
We round the sunny garden play,
Each little Indian sleepy-head
Is being kissed and put to bed.

And when at eve I rise from tea,
Day dawns beyond the Atlantic Sea;
And all the children in the West
Are getting up and being dressed.

What is the main idea of the poem “The Sun Travels”?

A. Not all parts of the world are on the same time.
B. The daily routine of an ordinary child can be mundane.
C. The patterns of the sun and earth are predictable.
D. Cultural differences lead to differences in the time of day when people eat and sleep.

3. The following is the poem “The Moon” by Robert Louis Stevenson:

The moon has a face like the clock in the hall;
She shines on thieves on the garden wall,
On streets and field and harbor quays,
And birdies asleep in the forks of the trees.

The squalling cat and the squeaking mouse,
The howling dog by the door of the house,
The bat that lies in bed at noon,
All love to be out by the light of the moon.

But all of the things that belong to the day
Cuddle to sleep to be out of her way;
And flowers and children close their eyes
Till up in the morning the sun shall arise.

Which of the following is the main idea of the poem “The Moon”?

A. The moon looks over the night and observes all the bad things that are awake when the sun sets.
B. Nighttime is just as interesting and active as the day, but it belongs to different actors.
C. The moon, like a kindly woman, oversees her own distinct world while the world of day sleeps.
D. Some animals are active during the day and some animals are active at night.

4. The following is the poem “The Swing” by Robert Louis Stevenson:

How do you like to go up in a swing,
Up in the air so blue?
Oh, I do think it the pleasantest thing
Ever a child can do!

Up in the air and over the wall,
Till I can see so wide,
Rivers and trees and cattle and all
Over the countryside—

Till I look down on the garden green,
Down on the roof so brown—
Up in the air I go flying again,
Up in the air and down!

What is the purpose of the poem “The Swing”?

A. To describe the great swing of emotions which is common in a small child.
B. To show a child’s delight in the simple pleasure of riding a common swing.
C. To illustrate the beautiful view that is visible from great heights.
D. To describe the physical sensation often felt when riding a swing.

­­­­­­­­­­­­­­­­5. A student, having read a poem by Robert Louis Stevenson, claims that the children in the poem are peering into a river as they would a mirror. Which quotation from the poem most effectively illustrates the claim?

A. Smooth it glides upon its travel/ Here a wimple, there a gleam/ O the glean gravel/ O the smooth stream.
B. We can see our colored faces/ Floating on the shaken pool/ Down in cool places/ Dim and very cool;
C. Patience, children, just a minute/ See the spreading circles die/ The stream and all in it/ Will clear by-and-by.
D.  Sailing blossoms, silver fishes/ Paven pools as clear as air/ How a child wishes/ To live down there!

6. The following is the poem “From a Railway Carriage” by Robert Louis Stevenson:

Faster than fairies, faster than witches,
Bridges and houses, hedges and ditches;
And charging along like troops in a battle
All through the meadows the horses and cattle:
All of the sights of the hill and the plain
Fly as thick as driving rain;
And ever again, in the wink of an eye,
Painted stations whistle by.
Here is a child who clambers and scrambles,
All by himself and gathering brambles;
Here is a tramp who stands and gazes;
And there is the green for stringing the daisies
Here is a cart run away in the road
Lumping along with man and load;
And here is a mill, and there is a river:
Each a glimpse and gone for ever!

Which choice best describes the function of the underlined portion in the text as a whole?

A. It describes the inability of the narrator to visualize the world around him.
B. It explains to the reader the energy of the child described later on in the poem.
C. It paints a picture of the “glimpse gone forever” that the narrator will later introduce.
D. It helps the reader realize that the poem is a description of the world from a fast-moving train.

7. The following is the poem “The Hayloft” by Robert Louis Stevenson:

Through all the pleasant meadow-side
The grass grew shoulder-high,
Till the shining scythes went far and wide
And cut it down to dry.

Those green and sweetly smelling crops
They led in wagons home;
And they piled them here in mountain tops
For mountaineers to roam.

Here is Mount Clear, Mount Rusty-Nail, Mount Eagle and Mount High;—
The mice that in these mountains dwell,
No happier are than I!

Oh, what a joy to clamber there,
Oh, what a place for play,
With the sweet, the dim, the dusty air,
The happy hills of hay!


According to the text, what is it that Mount Clear, Mont Rusty-Nail, Mount Eagle, and Mount High are made from?
A. The homes of mice.
B. The shoulder high grass, growing in the meadows
C. The imagination of a child
D. The dried hay.

8. “The Unseen Playmate” is a poem by Robert Louis Stevenson. Having read the poem to her class, a teacher claims that the playmate described in the poem is only present when children play by themselves. What quotation from “The Unseen Playmate” most effectively illustrates the claim?

A.  Nobody heard him and nobody saw/ His is a picture you never could draw.
B. Tis he that inhabits the caves that you dig/ ‘Tis he when you play with your soldiers of tin/ That sides with the Frenchmen and never can win.
C. When children are happy and lonely and good/ The Friend of the Children comes out of the wood.
D. When’er you are happy and cannot tell why/ The Friend of the Children is sure to be by!

9. “My Kingdom” is a poem written by Robert Louis Stevenson. After reading it, a class is confused as to whether the kingdom is literal. A student claims that it is figurative. Which quotation from “My Kingdom” most effectively illustrates the claim?

A. I played there were no deeper seas/ Nor any wider plains than these/ Nor other kings than me.
B. Down by a shining water well/ I found a very little dell
C. This was the world and I was king/ For me the bees came by to sing
D. How very big my nurse appeared/ How great and cool the rooms!

10.“My Treasures” is a poem by Robert Louis Stevenson. Having read it, a child claims that the narrator’s treasures are just pointless pieces of nature. Which quotation from “My Treasures” most effectively disproves this claim?

A. The stone, with the white and the yellow and grey/ We discovered I cannot tell how far away;
B. This whistle we made (and how clearly it sounds)/ By the side of a field at the end of the grounds/ of a branch of a plane, with a knife of my own
C. But of all my treasures the last is the king/ For there’s very few children possess such a thing/ And that is a chisel, both handle and blade.
D. And I carried it back, although weary and cold/ For though father denies it, I’m sure it is gold.

Answer Explanations

  1. A.  The poem tells of a child who, though sick, still explores a vast imaginary land complete with soldiers, ships, cities, and countryside. It is an exploration of imagination. The things described are not literal, so it is not answer B or C. The poem mentions physical malady only once, and it is not the main focus or purpose, so it is not D.
  2. A. In the poem, we read the reflections of a child who points out that as he or she does each of his or her daily tasks, other people in other parts of the world will be doing something else as it is different times of day in different places. This best fits with answer option A. Answer option B is incorrect as it does not address different parts of the world. Answer option C is incorrect because, while the earth and sun are mentioned, discussing them is not the main idea. Option D is incorrect as it is not because of cultural differences that people do things at different times, it is because of their varying geography.
  3. C. Notice how the poem personifies the moon, giving it a face, calling it “she”. In addition, the poem points out the normally active parts of the day that are quiet at night and vice versa. The poem is painting an image of a different world, that only comes to life under the watchful eye of the moon. This makes option C the best answer. Option A is incorrect as there is no evidence that there are many bad things at night (even though the poem mentions thieves). Option B is incorrect as it neglects to mention the moon. Option D is incorrect as it is a description of one thing the poem mentions, but fails to show the bigger main idea.
  4. B.  In the first stanza, the narrator says that riding a swing is “the pleasantest thing/ever a child can do”. He goes on to describe the pleasantness, joy, and delights of such a ride. This makes option B the best answer. The narrator does not describe the physical sensations that he feels, making option D incorrect. Emotions are not discussed, making option A incorrect. While the narrator does describe the incredible view, this is not the main purpose of the passage, but rather, just part of the evidence proving why he delights in riding a swing.
  5. B. Answer options A, C, and D all give evidence that the narrator or child is near a river, or even looking at a river; however, only option B gives evidence that they are using the river as a mirror. The evidence states that they can see their faces floating on the water. This makes B the best proof of the claim that they are “peering into a river as they would a mirror.”
  6. D. The underlined portion essentially says that “stations” are whizzing by “in the wink of an eye”. This is what leads the reader to realize that the narrator is on a train, traveling at quick speeds. This brings the rest of the poem into focus as the reader realizes it’s a description of all the things zipping past outside the windows. Without the underlined portion, the poem makes no sense. This makes option D the best answer. The other options don’t capture what the underlined portion is saying, but rather connect it to singular details elsewhere in the poem, not to the poem as a whole.
  7. D. We see in the poem that the shoulder-high grass is cut down (with scythes) and dried before being transported in wagons and then piled high into the mountains of hay on which the narrator plays. The grass is therefore not still growing in the meadows, making option B incorrect. The hills of hay contain mice, but are not made up of mice homes, making option A incorrect. The hills are literal stacks of hay, making option C incorrect.
  8. C. The teacher claims that the playmate only emerges when children play by themselves, or when they are alone. Option C clearly states that when children are lonely, the Friend of the Children (the unseen playmate) emerges. The other options all describe him in one way or another, but they don’t clarify that he only comes to play when a child is alone.
  9. A. A literal kingdom would be a country that is actually ruled over by a monarch. The question asks for evidence that this is not literal, but rather figurative, in other words, that the “kingdom” is not real. Answer option A best fits this description by showing that this is all play. The narrator is “playing” or pretending that he is the only king over deep seas and wide plains. Answer option B could be literal or figurative. Answer C has some figurative language in the bees singing, but does not prove that the narrator is not a literal king with a literal kingdom. Option D gives no evidence of a literal or figurative kingdom.
  10. C.  The question is a bit different in that is asks you to disprove a claim. We are therefore looking for evidence that not all the narrator’s treasures are pointless pieces of nature. Choice A describes a treasure which is a rock. Choice B describes a treasure which is a whistle made of a branch. Both these could be described as pointless pieces of nature. Choice C, however, is a man-made object. This, as a treasure, disproves the claim that all the treasures are pieces of nature.


If you’d like to read more poetry by Robert Louis Stevenson, you can explore his work on Project Gutenberg. Below is a link to the book from which each of these poems was taken.

https://www.gutenberg.org/cache/epub/25609/pg25609-images.html

General ACT and SAT Test-Taking Tips

  1. Schedule your test well in advance, and try to select a site that is connected to a high school. Sites that are not connected to high schools have less accountability to the students and tend to be more likely to cancel tests at the last moment. Taking the test at your high school is generally best. If your school isn’t a testing center, consider asking the administration why that is.
  2. Make sure to read your emails! The ACT and SAT will both email you occasionally with important information. It might be a reminder to upload your picture (which you can’t take the ACT without). It might be an update to your testing location (so you show up at the right place). It might be the notification to set up your SAT prior to arriving at your test center (you can’t do it once you arrive). Many students ignore these emails and then have big problems on test day.
  3. Have a regular schedule for practice and tutoring. Having a scheduled time in your week to practice or meet with a tutor will provide better results than intermittent or random practice squeezed into any free spot.
  4. Practice like you play. When doing practice at home, make sure you are giving it your all and treating it like test day. Practicing while lounging on the couch, eating a snack, and intermittently texting a friend will be far less effective than sitting at a desk or table, timing yourself, and focusing without interruption.
  5. Practice what you’re good at too. A lot of students make the mistake of only focusing on their weaknesses. It is often easy to improve what you’re already good at, so don’t forget to put effort into each section of your test.
  6. Don’t cut yourself slack. It’s very tempting to ignore small mistakes. “Oh, it was just a silly slip up” is easy to say. Small mistakes lead to wrong answers, just like big mistakes do, so treat a small mistake just as seriously as a big one. Identify why that small mistake happened and work towards eliminating what caused it.
  7. Sleep. High school students generally need over 8 hours of sleep each night. Just because you can exist on less doesn’t mean that is ideal. Best brain function occurs when students are well rested the week or two prior to the test. Start practicing healthy sleep habits today.

Old ACT vs. Digital SAT: Which One is Better for Me?

With the Digital SAT upon us, conventional advice about which test on which to focus has gone out the window. Students find themselves unsure as to which test is best for them. With colleges accepting either test, which one is best for you? Let’s take a look at the similarities and differences between the tests. Take a look at our info-graphic, and then read below to help decide which test is best for you or your student!

English, Reading, and Writing

On the ACT you’ll see a long English section that tests grammar, mechanics, and composition skills. There will also be a reading comprehension section later on in the test that requires that students read four long passages and answer 10 questions about each of them. On the Digital SAT these two sections have been combined into modules that cover both reading and writing. The reading passages are far shorter (max 150 words each) but the writing questions overlap many concepts covered on the ACT. The relative brevity of the passages on the SAT gives students with shorter focus periods a better chance at showing their skills. However, the constantly changing topics may be distracting.

Math

The ACT math tends to cover a wide range of topics from elementary school up through introductory pre-calculus. Students will need a broad understanding of many topics and the ability to do math quickly in order to do well on the ACT. The ACT focuses on testing simple concepts in diverse settings. The SAT covers fewer subjects, but does so more in depth. Students must have a much deeper understanding of algebra and linear geometry to succeed on the SAT.

Timing

The SAT, in general, is a deep, narrow test. The test expects students to have a thorough understanding of fewer concepts compared to the ACT which expects a shallower understanding of more concepts. For this reason, the ACT expects students to think quickly and adroitly while the SAT grants more time for deep thinking. Most students can expect to run out of time on at least one section of the ACT, while on the SAT this is less of a concern.

Adaptiveness

The SAT is now adaptive; the ACT is not. Depending on how a student does on the first reading and writing module and the first math module of the SAT, their second modules may be easier or harder. Consequently, the SAT can be shorter than the ACT and still collect a large amount of information on a student’s skills. It also means that students sitting next to one another will have different tests, thus reducing the risk of cheating.

Students who prefer the ACT

Students who prefer the ACT tend to be big readers and quick thinkers. Students who read a lot in their free time (or who did in the past) tend to have an advantage in terms of speed and skill on the ACT. In addition, students who are good with data and scientific concepts will have an advantage on the science portion of the ACT. Students who receive extended time or other accommodations often prefer the ACT as well.

Students who prefer the SAT

Students who are strong in math (especially Algebra) tend to do well on the SAT. Students who prefer to have more time to think deeply about concepts, wording, and nuance also tend to prefer the SAT. Students with a shorter attention span, will often prefer the shorter passages and more direct wording of the questions on the reading and writing portion of the SAT.

The Long and the Short of It

If by now it isn’t obvious which test you should focus on, consider taking one of each to compare. Nothing beats the real-world experience of giving it a try.  If you’re having a hard time making a decision based on your scores and skills, please reach out: we’re always happy to help!